LIPINCOTT Antihypertensives Flashcards

1
Q

17.1 A 45-year-old man was just started on therapy for
hypertension and developed a persistent, dry cough.
Which is most likely responsible for this side effect?
A. Enalapril.
B. Losartan.
C. Nifedipine.
D. Prazosin.
E. Propranolol.

A

Correct answer = A. The cough is most likely an adverse
effect of the ACE inhibitor enalapril. Losartan is an ARB
that has the same beneficial effects as an ACE inhibitor but
is less likely to produce a cough. Nifedipine, prazosin, and
propranolol do not cause this side effect

How well did you know this?
1
Not at all
2
3
4
5
Perfectly
2
Q

17.2 Which may cause reflex tachycardia and/or postural
hypotension on initial administration?
A. Atenolol.
B. Hydrochlorothiazide.
C. Metoprolol.
D. Prazosin.
E. Verapamil.

A

Correct answer = D. Prazosin produces first-dose hypotension,
presumably by blocking α1 receptors. This effect is
minimized by initially giving the drug in small, divided doses.
The other agents do not have this adverse effect.

How well did you know this?
1
Not at all
2
3
4
5
Perfectly
3
Q

17.3 Which can precipitate a hypertensive crisis following
abrupt cessation of therapy?
A. Clonidine.
B. Diltiazem.
C. Enalapril.
D. Losartan.
E. Hydrochlorothiazide.

A

Correct answer = A. Increased sympathetic nervous system
activity occurs if clonidine therapy is abruptly stopped after
prolonged administration. Uncontrolled elevation in blood
pressure can occur. Patients should be slowly weaned
from clonidine while other antihypertensive medications
are initiated. The other drugs on the list do not produce this
phenomenon.

How well did you know this?
1
Not at all
2
3
4
5
Perfectly
4
Q

17.4 A 48-year-old hypertensive patient has been
successfully treated with a thiazide diuretic for the last 5
years. Over the last 3 months, his diastolic pressure has
steadily increased, and he was started on an additional
antihypertensive agent. He complains of several
instances of being unable to achieve an erection and
not being able to complete three sets of tennis as he
once did. Which is the likely second antihypertensive
medication?
A. Captopril.
B. Losartan.
C. Metoprolol.
D. Minoxidil.
E. Nifedipine

A

Correct answer = C. The side effect profile of β-blockers,
such as metoprolol, is characterized by interference with
sexual performance and decreased exercise tolerance.
None of the other drugs is likely to produce this combination
of side effects.

How well did you know this?
1
Not at all
2
3
4
5
Perfectly
5
Q

17.5 A 40-year-old male has recently been diagnosed with
hypertension due to pressure readings of 163/102
and 165/100 mm Hg. He also has diabetes that is well
controlled with oral hypoglycemic medications. Which
is the best initial treatment regimen for treatment of
hypertension in this patient?
A. Felodipine.
B. Furosemide.
C. Lisinopril.
D. Lisinopril and hydrochlorothiazide.
E. Metoprolol.

A

Correct answer = D. Because the systolic blood pressure
is more than 20 mm Hg above goal (10 mm Hg above goal
diastolic), treatment with two different medications is preferred.
Because the patient is diabetic, he also has a compelling
indication for an ACE inhibitor or ARB.

How well did you know this?
1
Not at all
2
3
4
5
Perfectly
6
Q

17.6 A 60-year-old white female has not reached her blood
pressure goal after 1 month of treatment with a low dose
of lisinopril. All of the following would be appropriate
next steps in the treatment of her hypertension except:
A. Increase dose of lisinopril.
B. Add a diuretic medication.
C. Add on a calcium channel blocker medication.
D. Add on an ARB medication.

A

Correct answer = D. Increasing the dose of lisinopril or adding
a second medication from a different class (such as a
calcium channel blocker or diuretic) would be appropriate
steps to control the blood pressure. Adding an ARB as the
second medication is not recommended. ARBs have a similar
mechanism of action to ACE inhibitors, and combination
therapy may increase the risk of adverse effects.

How well did you know this?
1
Not at all
2
3
4
5
Perfectly
7
Q

17.7 A patient returns to her health care provider for routine
monitoring 3 months after her hypertension regimen
was modified. Labs reveal elevated serum potassium.
Which is likely responsible for this hyperkalemia?
A. Chlorthalidone.
B. Clonidine.
C. Furosemide.
D. Losartan.
E. Nifedipine.

A

Correct answer = D. Losartan, an ARB, can cause an
increase in serum potassium similar to ACE inhibitors.
Furosemide and chlorthalidone can cause a decrease in
serum potassium. Nifedipine and clonidine do not affect
potassium levels

How well did you know this?
1
Not at all
2
3
4
5
Perfectly
8
Q

17.7 A patient returns to her health care provider for routine
monitoring 3 months after her hypertension regimen
was modified. Labs reveal elevated serum potassium.
Which is likely responsible for this hyperkalemia?
A. Chlorthalidone.
B. Clonidine.
C. Furosemide.
D. Losartan.
E. Nifedipine.

A

Correct answer = D. Losartan, an ARB, can cause an
increase in serum potassium similar to ACE inhibitors.
Furosemide and chlorthalidone can cause a decrease in
serum potassium. Nifedipine and clonidine do not affect
potassium levels

How well did you know this?
1
Not at all
2
3
4
5
Perfectly
9
Q

17.8 A 58-year-old female reports that she recently stopped
taking her blood pressure medications because of
swelling in her feet that began shortly after she started
treatment. Which is most likely to cause peripheral
edema?
A. Atenolol.
B. Clonidine.
C. Felodipine.
D. Hydralazine.
E. Prazosin

A

Correct answer = C. Peripheral edema is one of the most
common side effects of calcium channel blockers. None of
the other agents commonly cause peripheral edema.

How well did you know this?
1
Not at all
2
3
4
5
Perfectly
10
Q

17.9 Which is an appropriate choice for hypertension
treatment during pregnancy?
A. Aliskiren.
B. Fosinopril.
C. Hydralazine.
D. Valsartan.

A

Correct answer = C. Hydralazine is an appropriate choice
for a hypertensive pregnant patient. ACE inhibitors, ARBs,
and the direct renin inhibitor, aliskiren, are all contraindicated
in pregnancy due to their potential for fetal harm

How well did you know this?
1
Not at all
2
3
4
5
Perfectly
11
Q

18.1 An elderly patient with a history of heart disease is
brought to the emergency room with difficulty breathing.
Examination reveals that she has pulmonary edema.
Which treatment is indicated?
A. Acetazolamide.
B. Chlorthalidone.
C. Furosemide.
D. Hydrochlorothiazide.
E. Spironolactone.

A

Correct answer = C. This is a potentially fatal situation. It
is important to administer a diuretic that will reduce fluid
accumulation in the lungs and, thus, improve oxygenation
and heart function. The loop diuretics are most effective in
removing large fluid volumes from the body and are the
treatment of choice in this situation. In this situation, furosemide
should be administered intravenously. The other
choices are inappropriate.

How well did you know this?
1
Not at all
2
3
4
5
Perfectly
12
Q

18.2 A group of college students is planning a mountain
climbing trip to the Andes. Which would be appropriate
for them to take to prevent mountain sickness?
A. A thiazide diuretic such as hydrochlorothiazide.
B. An anticholinergic such as atropine.
C. A carbonic anhydrase inhibitor such as
acetazolamide.
D. A loop diuretic such as furosemide.
E. A β-blocker such as metoprolol.

A

Correct answer = C. Acetazolamide is used prophylactically
for several days before an ascent above 10,000 feet. This
treatment prevents the cerebral and pulmonary problems
associated with the syndrome as well as other difficulties,
such as nausea.

How well did you know this?
1
Not at all
2
3
4
5
Perfectly
13
Q

18.3 An alcoholic male has developed hepatic cirrhosis.
To control the ascites and edema, which should be
prescribed?
A. Acetazolamide.
B. Chlorthalidone.
C. Furosemide.
D. Hydrochlorothiazide.
E. Spironolactone.

A

Correct answer = E. Spironolactone is very effective in the
treatment of hepatic edema. These patients are frequently
resistant to the diuretic action of loop diuretics, although
a combination with spironolactone may be beneficial. The
other agents are not indicated.

How well did you know this?
1
Not at all
2
3
4
5
Perfectly
14
Q

18.4 A 55-year-old male with kidney stones has been placed
on a diuretic to decrease calcium excretion. However,
after a few weeks, he develops an attack of gout. Which
diuretic was he taking?
A. Furosemide.
B. Hydrochlorothiazide.
C. Spironolactone.
D. Triamterene.
E. Urea.

A

Correct answer = B. Hydrochlorothiazide is effective in
increasing calcium reabsorption, thus decreasing the amount
of calcium excreted, and decreasing the formation of kidney
stones that contain calcium phosphate or calcium oxalate.
However, hydrochlorothiazide can also inhibit the excretion of
uric acid and cause its accumulation, leading to an attack of
gout in some individuals. Furosemide increases the excretion
of calcium, whereas the K+-sparing osmotic diuretics, spironolactone
and triamterene, and urea do not have an effect.

How well did you know this?
1
Not at all
2
3
4
5
Perfectly
15
Q

18.5 A 75-year-old woman with hypertension is being treated
with a thiazide. Her blood pressure responds well and
reads at 120/76 mm Hg. After several months on the
medication, she complains of being tired and weak. An
analysis of the blood indicates low values for which of
the following?
A. Calcium.
B. Glucose.
C. Potassium.
D. Sodium.
E. Uric acid.

A

Correct answer = C. Hypokalemia is a common adverse
effect of the thiazides and causes fatigue and lethargy in the
patient. Supplementation with potassium chloride or foods
high in K+ corrects the problem. Alternatively, a potassiumsparing
diuretic, such as spironolactone, may be added.
Calcium, uric acid, and glucose are usually elevated by thiazide
diuretics. Sodium loss would not weaken the patient.

How well did you know this?
1
Not at all
2
3
4
5
Perfectly
16
Q

18.6 Which is contraindicated in a patient with hyperkalemia?
A. Acetazolamide.
B. Chlorthalidone.
C. Chlorothiazide.
D. Ethacrynic acid.
E. Spironolactone.

A

Correct answer = E. Spironolactone acts in the collecting
tubule to inhibit Na+ reabsorption and K+ excretion. It
is extremely important that patients who are treated with
any potassium-sparing diuretic be closely monitored for
potassium levels. Exogenous potassium supplementation
is usually discontinued when potassium-sparing diuretic
therapy is instituted and spironolactone is contraindicated
in patients with hyperkalemia. The other drugs promote the
excretion of potassium.

How well did you know this?
1
Not at all
2
3
4
5
Perfectly
17
Q

18.7 Which of the following should be avoided in a patient
with a history of severe anaphylactic reaction to sulfa
medications?
A. Amiloride.
B. Hydrochlorothiazide.
C. Mannitol.
D. Spironolactone.
E. Triamterene.

A

Correct answer = B. Hydrochlorothiazide, like many thiazide
and thiazide-like diuretics, contains a sulfa moiety within its
chemical structure. It is important to avoid use in those individuals
with severe hypersensitivity to sulfa medications. It
may be used with caution, however, in those with only minor
reaction to sulfa medications.

How well did you know this?
1
Not at all
2
3
4
5
Perfectly
18
Q

18.8 A male patient is placed on a new medication and
notes that his breasts have become enlarged and
tender to the touch. Which medication is he most likely
taking?
A. Chlorthalidone.
B. Furosemide.
C. Hydrochlorothiazide.
D. Spironolactone.
E. Triamterene.

A

Correct answer = D. An adverse drug reaction to spironolactone
is gynecomastia due to its effects on androgens
and progesterone in the body. Eplerenone may be a suitable
alternative if the patient is in need of an aldosterone
antagonist but has a history of gynecomastia.

How well did you know this?
1
Not at all
2
3
4
5
Perfectly
19
Q

18.9 A patient presents to the emergency department with
an extreme headache. After a thorough workup, the
attending physician concludes that the pain is due to
increased intracranial pressure. Which diuretic would
work best to reduce this pressure?
A. Acetazolamide.
B. Indapamide.
C. Furosemide.
D. Hydrochlorothiazide.
E. Mannitol.

A

Correct answer = E. Osmotic diuretics, such as mannitol,
are a mainstay of treatment for patients with increased
intracranial pressure or acute renal failure due to shock,
drug toxicities, and trauma.

How well did you know this?
1
Not at all
2
3
4
5
Perfectly
20
Q

18.10 Which diuretic has been shown to improve blood
pressure in resistant hypertension or those already
treated with three blood pressure medications
including a thiazide or thiazide-like diuretic?
A. Chlorthalidone.
B. Indapamide.
C. Furosemide.
D. Mannitol.
E. Spironolactone.

A

Correct answer = E. Resistant hypertension, defined by
the use of three or more medications without reaching the
blood pressure goal, often responds well to aldosterone
antagonists. This effect can be seen in those with or without
elevated aldosterone levels.

How well did you know this?
1
Not at all
2
3
4
5
Perfectly
21
Q

19.1 Which drug may exacerbate HF?
A. Acetaminophen.
B. Cetirizine.
C. Chlorthalidone.
D. Ibuprofen.

A

Correct answer = D. NSAIDs, such as ibuprofen, lead to
increased fluid retention and increased blood pressure. If
possible, NSAIDs should be avoided in HF patients in order
to avoid exacerbations of HF.

How well did you know this?
1
Not at all
2
3
4
5
Perfectly
22
Q

19.2 Which best describes the action of ACE inhibitors on
the failing heart?
A. ACE inhibitors increase vascular resistance.
B. ACE inhibitors decrease cardiac output.
C. ACE inhibitors reduce preload.
D. ACE inhibitors increase aldosterone.

A

Correct answer = C. ACE inhibitors decrease vascular resistance,
decrease preload, decrease afterload, and increase
cardiac output. In addition, ACE inhibitors blunt aldosterone
release.

How well did you know this?
1
Not at all
2
3
4
5
Perfectly
23
Q

19.3 What makes losartan different from other ARBs?
A. Losartan is renally eliminated.
B. Losartan has an active metabolite.
C. Losartan has the shortest half-life.
D. Losartan has a small volume of distribution

A

Correct answer = B. Losartan is the only ARB that undergoes
first-pass metabolism to convert to its active metabolite.
Most ARBs have once-daily dosing, and all (except
candesartan) have large volumes of distribution

How well did you know this?
1
Not at all
2
3
4
5
Perfectly
24
Q

19.4 How do β-blockers improve cardiac function in HF?
A. By decreasing cardiac remodeling.
B. By increasing heart rate.
C. By increasing renin release.
D. By activating norepinephrine

A

Correct answer = A. Although it seems counterintuitive to
decrease heart rate in HF, β-blockers improve cardiac functioning
by slowing heart rate, decreasing renin release, and
preventing the direct effects of norepinephrine on cardiac
muscle to decrease remodeling

How well did you know this?
1
Not at all
2
3
4
5
Perfectly
25
Q

19.5 BC is a 70-year-old female who is diagnosed with
HFrEF. Her past medical history is significant for
hypertension and atrial fibrillation. She is taking
hydrochlorothiazide, lisinopril, metoprolol tartrate, and
warfarin. BC says she is feeling “good” and has no
cough, shortness of breath, or edema. Which is the
most appropriate medication change to make?
A. Discontinue hydrochlorothiazide.
B. Change lisinopril to losartan.
C. Decrease warfarin dose.
D. Change metoprolol tartrate to metoprolol succinate.

A

Correct answer = D. Metoprolol succinate should be used in
HF, given that there is mortality benefit shown with metoprolol
succinate in landmark HF trials. Hydrochlorothiazide and
warfarin are appropriate based on the information given;
there is no reason to change to an ARB since the patient
has no cough or history of angioedema

How well did you know this?
1
Not at all
2
3
4
5
Perfectly
26
Q

19.6 SC is a 75-year-old white male who has HF. He is seen
in clinic today, reporting shortness of breath, increased
pitting edema, and a 5-pound weight gain over the
last 2 days. His current medication regimen includes
losartan and metoprolol succinate. SC has no chest
pain and is deemed stable for outpatient treatment.
Which of the following is the best recommendation?
A. Increase the dose of metoprolol succinate.
B. Start hydrochlorothiazide.
C. Start furosemide.
D. Discontinue losartan.

A

Correct answer = C. As it is possible that SC is having a
HF exacerbation, increasing the dose of the β-blocker is not
indicated at this time. There is no reason to stop losartan,
based on the information we have. Loop diuretics are preferred
over thiazide diuretics when patients require diuresis
immediately.

How well did you know this?
1
Not at all
2
3
4
5
Perfectly
27
Q

19.7 How is spironolactone beneficial in HF?
A. Promotes potassium secretion.
B. Agonizes aldosterone.
C. Prevents cardiac hypertrophy.
D. Decreases blood glucose.

A

Correct answer = C. Spironolactone antagonizes aldosterone,
which in turn prevents salt/water retention, cardiac
hypertrophy, and hypokalemia. Spironolactone has endocrine
effects on hormones but not on glucose.

How well did you know this?
1
Not at all
2
3
4
5
Perfectly
28
Q

19.8 Which is important to monitor in patients taking
digoxin?
A. Chloride.
B. Potassium.
C. Sodium.
D. Zinc.

A

Correct answer = B. Hypokalemia can lead to life-threatening
arrhythmias and increases the potential of cardiac toxicity
with digoxin.

How well did you know this?
1
Not at all
2
3
4
5
Perfectly
29
Q

19.9 Which describes the mechanism of action of milrinone
in HF?
A. Decreases intracellular calcium.
B. Increases cardiac contractility.
C. Decreases cAMP.
D. Activates phosphodiesterase.

A

Correct answer = B. Milrinone is a phosphodiesterase inhibitor
that leads to increased cAMP, increased intracellular
calcium, and therefore increased contractility.

30
Q

19.10 What is the most common adverse effect associated
with fixed-dose hydralazine/isosorbide dinitrate?
A. Diarrhea.
B. Drug-induced lupus.
C. Headache.
D. Heartburn.

A

Correct answer = C. While drug-induced lupus is a possibility
with hydralazine, headache is the most common adverse
effect

31
Q

20.1 A 60-year-old woman had a myocardial infarction.
Which of the following should be used to prevent
life-threatening arrhythmias that can occur post–
myocardial infarction in this patient?
A. Digoxin.
B. Flecainide.
C. Metoprolol.
D. Procainamide.
E. Quinidine.

A

Correct answer = C. β-Blockers such as metoprolol prevent
arrhythmias that occur subsequent to a myocardial infarction.
None of the other drugs has been shown to be effective
in preventing postinfarct arrhythmias. Flecainide should
be avoided in patients with structural heart disease

32
Q

20.2 Suppression of arrhythmias resulting from a reentry
focus is most likely to occur if the drug:
A. Has vagomimetic effects on the AV node.
B. Is a β-blocker.
C. Converts a unidirectional block to a bidirectional
block.
D. Slows conduction through the atria.
E. Has atropine-like effects on the AV node.

A

Correct answer = C. Current theory holds that a reentrant
arrhythmia is caused by damaged heart muscle, so that
conduction is slowed through the damaged area in only one
direction. A drug that prevents conduction in either direction
through the damaged area interrupts the reentrant arrhythmia.
Class I antiarrhythmics, such as lidocaine, are capable
of producing bidirectional block. The other choices do not
have any direct effects on the direction of blockade of conduction
through damaged cardiac muscle.

33
Q

20.3 A 57-year-old man is being treated for an atrial
arrhythmia. He complains of dry mouth, blurred vision,
and urinary hesitancy. Which antiarrhythmic drug is he
mostly like taking?
A. Metoprolol.
B. Disopyramide.
C. Dronedarone.
D. Sotalol.

A

Correct answer = B. The clustered symptoms of dry mouth,
blurred vision, and urinary hesitancy are characteristic of
anticholinergic adverse effects which are caused by class
IA agents (in this case, disopyramide). The other drugs do
not cause anticholinergic effects.

34
Q

20.4 A 58-year-old woman is being treated for chronic
suppression of a ventricular arrhythmia. After 1 week
of therapy, she complains about feeling severe upset
stomach and heartburn. Which antiarrhythmic drug is
the likely cause of these symptoms?
A. Amiodarone.
B. Digoxin.
C. Mexiletine.
D. Propranolol.
E. Quinidine.

A

Correct answer = C. The patient is exhibiting a classic
adverse effect of mexiletine. None of the other agents listed
are likely to cause dyspepsia.

35
Q

20.5 A 78-year-old woman has been newly diagnosed with
atrial fibrillation. She is not currently having symptoms
of palpitations or fatigue. Which is appropriate to
initiate for rate control as an outpatient?
A. Amiodarone.
B. Dronedarone.
C. Esmolol.
D. Flecainide.
E. Metoprolol.

A

Correct answer = E. Only C and E are options to control
rate. The other options are used for rhythm control in
patients with atrial fibrillation. Since esmolol is IV only, the
only option to start as an outpatient is metoprolol.

36
Q

20.6 Which of the following is correct regarding digoxin
when used for atrial fibrillation?
A. Digoxin works by blocking voltage-sensitive
calcium channels.
B. Digoxin is used for rhythm control in patients with
atrial fibrillation.
C. Digoxin increases conduction velocity through the
AV node.
D. Digoxin levels of 1 to 2 ng/mL are desirable in the
treatment of atrial fibrillation.

A

Correct answer = D. Digoxin works by inhibiting the Na+/K+-
ATPase pump. It decreases conduction velocity through the
AV node and is used for rate control in atrial fibrillation (not
rhythm control). Digoxin levels between 1 and 2 ng/mL are
more likely to exhibit negative chronotropic effects desired
in atrial fibrillation or flutter. A serum drug concentration
between 0.5 and 0.8 ng/mL is for symptomatic management
of heart failure.

37
Q

20.7 All of the following are adverse effects of amiodarone
except:
A. Cinchonism.
B. Hypothyroidism.
C. Hyperthyroidism.
D. Pulmonary fibrosis.
E. Blue skin discoloration.

A

Correct answer = A. Cinchonism is a constellation of symptoms
(blurred vision, tinnitus, headache, psychosis) that is
known to occur with quinidine. All other options are adverse
effects with amiodarone that require close monitoring.

38
Q

20.8 Which arrhythmia can be treated with lidocaine?
A. Paroxysmal supraventricular ventricular
tachycardia.
B. Atrial fibrillation.
C. Atrial flutter.
D. Ventricular tachycardia

A

Correct answer = D. Lidocaine has little effect on atrial or AV
nodal tissue; thus, it used for ventricular arrhythmias such
as ventricular tachycardia

39
Q

20.9 A clinician would like to initiate a drug for rhythm control
of atrial fibrillation. Which of the following coexisting
conditions would allow for initiation of flecainide?
A. Hypertension.
B. Left ventricular hypertrophy.
C. Coronary artery disease.
D. Heart failure.

A

Correct answer = A. Since flecainide can increase the risk
of sudden cardiac death in those with a history of structural
heart disease, only A will allow for flecainide initiation.
Structural heart disease includes left ventricular hypertrophy,
heart failure, and atherosclerotic heart disease.

40
Q

20.10 Which statement regarding dronedarone is correct?
A. Dronedarone is more effective than amiodarone.
B. QT interval prolongation is not a risk with
dronedarone.
C. Dronedarone increases the risk of death in
patients with permanent atrial fibrillation or
symptomatic heart failure.
D. There is no need to monitor liver function with
dronedarone

A

Correct answer = C. Dronedarone is not as effective as amiodarone,
QT prolongation is a risk with this drug, and liver
function should be monitored when taking dronedarone
since it increases the risk of liver failure. The drug is contraindicated
in those with symptomatic heart failure or permanent
atrial fibrillation due to an increased risk of death.

41
Q

21.1 What is the clinical term for angina caused by coronary
vasospasm?
A. Classic angina.
B. Myocardial infarction.
C. Prinzmetal angina.
D. Unstable angina.

A

Correct answer = C. Prinzmetal angina is angina caused
by vasospasm of the coronary arteries. It is also known
as vasospastic or variant angina. The other answers
refer to angina (with varying levels of severity) caused by
atherosclerosis.

42
Q

21.2 All of the following medications can be useful for
managing stable angina in a patient with coronary
artery disease except:
A. Amlodipine.
B. Atenolol.
C. Immediate-release nifedipine.
D. Isosorbide dinitrate.

A

Correct answer = C. The short-acting dihydropyridine calcium
channel blocker nifedipine should be avoided in CAD
patients as this can worsen angina; however, the extendedrelease
formulation can be used.

43
Q

21.3 A 72-year-old male presents to the primary care
clinic complaining of chest tightness and pressure
that is increasing in severity and frequency. His
current medications include atenolol, lisinopril, and
nitroglycerin. Which intervention is most appropriate at
this time?
A. Add amlodipine.
B. Initiate isosorbide mononitrate.
C. Initiate ranolazine.
D. Refer the patient to the nearest emergency room
for evaluation.

A

Correct answer = D. Crescendo angina is indicative of
unstable angina that requires further workup.

44
Q

21.4 A 62-year-old patient with a history of asthma and
vasospastic angina states that he gets chest pain both
with exertion and at rest, about ten times per week.
One sublingual nitroglycerin tablet always relieves
his symptoms, but this medication gives him an awful
headache every time he takes it. Which is the best
option for improving his angina?
A. Change to sublingual nitroglycerin spray.
B. Add amlodipine.
C. Add propranolol.
D. Replace nitroglycerin with ranolazine.

A

Correct answer = B. Calcium channel blockers are preferred
for vasospastic angina. β-Blockers can actually worsen
vasospastic angina; furthermore, nonselective β-blockers
should be avoided in patients with asthma. The nitroglycerin
spray would also be expected to cause headache, so this is
not the best choice. Ranolazine is not indicated for immediate
relief of an angina attack, nor is it a first-line option

44
Q

21.4 A 62-year-old patient with a history of asthma and
vasospastic angina states that he gets chest pain both
with exertion and at rest, about ten times per week.
One sublingual nitroglycerin tablet always relieves
his symptoms, but this medication gives him an awful
headache every time he takes it. Which is the best
option for improving his angina?
A. Change to sublingual nitroglycerin spray.
B. Add amlodipine.
C. Add propranolol.
D. Replace nitroglycerin with ranolazine.

A

Correct answer = B. Calcium channel blockers are preferred
for vasospastic angina. β-Blockers can actually worsen
vasospastic angina; furthermore, nonselective β-blockers
should be avoided in patients with asthma. The nitroglycerin
spray would also be expected to cause headache, so this is
not the best choice. Ranolazine is not indicated for immediate
relief of an angina attack, nor is it a first-line option

45
Q

21.5 Which side effect is associated with amlodipine?
A. Bradycardia.
B. Cough.
C. Edema.
D. QT prolongation

A

Correct answer = C. Edema is the correct answer. The other
answers are incorrect.

46
Q

21.6 Which medication should be prescribed to all anginal
patients to treat an acute attack?
A. Isosorbide dinitrate.
B. Nitroglycerin patch.
C. Nitroglycerin sublingual tablet or spray.
D. Ranolazine.

A

Correct answer = C. The other options will not provide
prompt relief of angina and should not be used to treat an
acute attack.

47
Q

21.7 A 65-year-old male experiences uncontrolled angina
attacks that limit his ability to do household chores. He
is adherent to a maximized dose of β-blocker with a low
heart rate and low blood pressure. He was unable to
tolerate an increase in isosorbide mononitrate due to
headache. Which is the most appropriate addition to his
antianginal therapy?
A. Amlodipine.
B. Aspirin.
C. Ranolazine.
D. Verapamil.

A

Correct answer = C. Ranolazine is the best answer. The
patient’s blood pressure is low, so verapamil and amlodipine
may drop blood pressure further. Verapamil may also
decrease heart rate. Ranolazine can be used when other
agents are maximized, especially when blood pressure is
well controlled. The patient will need a baseline ECG and
lab work to ensure safe use of this medication

48
Q

21.9 Which of the following medications would be safe to
use in a patient taking ranolazine?
A. Carbamazepine.
B. Clarithromycin.
C. Enalapril.
D. Quetiapine

A

Correct answer = C. All other medications should be avoided
due to potential drug–drug interactions

49
Q

21.10 A patient whose angina was previously well controlled
with once-daily isosorbide mononitrate states that
recently he has been taking isosorbide mononitrate
twice a day to control angina symptoms that are
occurring more frequently during early morning
hours. Which of the following is the best option for this
patient?
A. Continue once-daily administration of isosorbide
mononitrate but advise the patient to take this
medication in the evening.
B. Advise continuation of isosorbide mononitrate
twice daily for full 24-hour coverage of anginal
symptoms.
C. Switch to isosorbide dinitrate, as this has a longer
duration of action than the mononitrate.
D. Switch to nitroglycerin patch for consistent drug
delivery and advise him to wear the patch around
the clock

A

Correct answer = A. It is important to maintain a nitrate-free
period to prevent the development of tolerance to nitrate
therapy. The mononitrate formulation has the longer halflife.
The nitroglycerin patch should be taken off for 10 to 12
hours daily to allow for nitrate-free interval

50
Q

Choose the ONE best answer.
22.1 Which of the P2Y12 ADP receptor antagonists reversibly
binds the receptor?
A. Clopidogrel.
B. Prasugrel.
C. Ticagrelor.
D. Ticlopidine.

A

Correct answer = C. Of the four P2Y12 ADP receptor antagonists,
ticagrelor is the only one that reversibly binds the
receptor. This is important when it comes to compliance.
If a patient is not compliant, then the antiplatelet activity of
ticagrelor stops when the drug is missed (since the platelets
inhibited are not irreversibly inhibited as they would be with
aspirin, clopidogrel, or prasugrel). On the other hand, the
waiting period prior to surgery may be shorter in patients
taking ticagrelor since it takes less time for the antiplatelet
effect to wear off

51
Q

22.2 A 70-year-old female is diagnosed with nonvalvular
atrial fibrillation. Her past medical history is significant
for chronic kidney disease, and her renal function is
moderately diminished. All of the following anticoagulants
would be expected to require a reduced dosage in this
patient except:
A. Apixaban.
B. Dabigatran.
C. Rivaroxaban.
D. Warfarin.

A

Correct answer = D. Warfarin does not require dosage
adjustment in renal dysfunction. The INR is monitored and
dosage adjustments are made on the basis of this information.
All of the other agents are renally cleared to some
extent and require dosage adjustments in renal dysfunction

52
Q

22.3 An 80-year-old male is taking warfarin indefinitely for
the prevention of deep venous thrombosis. He is a
compliant patient with a stable INR and has no issues
with bleeding or bruising. He is diagnosed with a urinary
tract infection and is prescribed sulfamethoxazole/
trimethoprim. What effect will this have on his warfarin
therapy?
A. Sulfamethoxazole/trimethoprim will decrease the
anticoagulant effect of warfarin.
B. Sulfamethoxazole/trimethoprim will increase the
anticoagulant effect of warfarin.
C. Sulfamethoxazole/trimethoprim will activate platelet
activity.
D. Sulfamethoxazole/trimethoprim will not change
anticoagulation status.

A

Correct answer = B. Sulfamethoxazole/trimethoprim has
a significant drug interaction with warfarin, such that it will
inhibit warfarin metabolism. Therefore, sulfamethoxazole/
trimethoprim will cause increased anticoagulation, and the
patient will need to have his warfarin dose decreased and
INR checked frequently while he is on this antibiotic

53
Q

22.4 In which disease state is cilostazol contraindicated?
A. Peripheral arterial disease.
B. Gout.
C. Heart failure with reduced ejection fraction.
D. Osteoporosis.

A

Correct answer = C. Cilostazol is contraindicated in heart
failure with reduced ejection fraction because it is a phosphodiesterase
inhibitor and acts as a positive inotrope
(which can lead to sudden cardiac death).

54
Q

22.5 Which must heparin bind to in order to exert its
anticoagulant effect?
A. GP IIb/IIIa receptor.
B. Thrombin.
C. Antithrombin III.
D. von Willebrand factor

A

Correct answer = C. Heparin binds to antithrombin III, causing
a conformational change. This heparin/antithrombin III
complex then inactivates thrombin and factor Xa.

55
Q

22.6 Which is considered “fibrin selective” because it rapidly
activates plasminogen that is bound to fibrin?
A. Alteplase.
B. Fondaparinux.
C. Streptokinase.
D. Urokinas

A

Correct answer = A. Alteplase has a low affinity for free plasminogen
in the plasma, but it rapidly activates plasminogen
that is bound to fibrin in a thrombus or a hemostatic plug.
It has the advantage of lysing only fibrin, without unwanted
degradation of other proteins (notably fibrinogen

56
Q

22.7 A 56-year-old man presents to the emergency room
with complaints of swelling, redness, and pain in his
right leg. The patient is diagnosed with acute DVT and
requires treatment with an anticoagulant. All of the
following are approved for treatment of this patient’s
DVT except:
A. Rivaroxaban.
B. Dabigatran.
C. Enoxaparin.
D. Heparin

A

Correct answer = B. Dabigatran is only approved for the
prevention of stroke in nonvalvular atrial fibrillation; it is not
approved for the treatment of acute DVT. All of the other
options are approved for treatment of acute DVT.

57
Q

22.8 Which is most appropriate for reversing the
anticoagulant effects of heparin?
A. Aminocaproic acid.
B. Protamine sulfate.
C. Vitamin K1.
D. Tranexamic acid

A

Correct answer = B. Excessive bleeding may be managed
by ceasing administration of heparin or by treating with protamine
sulfate. Infused slowly, protamine sulfate combines
ionically with heparin to form a stable, inactive complex.
Aminocaproic acid and tranexamic acid are approved for
the treatment of hemorrhage but do not specifically reverse
the effects of heparin to stop bleeding. Vitamin K1 is used to
help reverse the effects of warfarin-induced bleeding

58
Q

22.9 A 62-year-old male taking warfarin for stroke prevention
in atrial fibrillation presents to his primary care physician
with an elevated INR of 10.5 without bleeding. He is
instructed to hold his warfarin dose and given 2.5 mg of
oral vitamin K1. When would the effects of vitamin K on
the INR most likely be noted in this patient?
A. 1 hour.
B. 6 hours.
C. 24 hours.
D. 72 hours.

A

Correct answer = C. Vitamin K1 takes about 24 hours to see
a reduction in the INR. This is due to the time required for
the body to synthesize new coagulation factors.

59
Q

22.10 A 58-year-old man receives intravenous alteplase
treatment for acute stroke. Five minutes following
completion of alteplase infusion, he develops orolingual
angioedema. Which of the following drugs may have
increased the risk of developing orolingual angioedema
in this patient?
A. ACE inhibitor.
B. GP IIb/IIIa receptor antagonist.
C. Phosphodiesterase inhibitor.
D. Thiazide diuretic.

A

Correct answer = A. ACE inhibitors, aspirin, and prasugrel
all have possible adverse effects including orolingual
angioedema. In the setting of alteplase administration, ACE
inhibitors have been associated with an increased risk of
developing orolingual angioedema with concomitant use.

60
Q

23.1 Which one of the following is the most common side
effect of antihyperlipidemic drug therapy?
A. Elevated blood pressure.
B. Gastrointestinal disturbance.
C. Neurologic problems.
D. Heart palpitations.
E. Migraine headaches

A

Correct answer = B. Gastrointestinal disturbances frequently
occur as a side effect of antihyperlipidemic drug
therapy. The other choices are not seen as commonly.

61
Q

23.2 Which one of the following hyperlipidemias is
characterized by elevated plasma levels of chylomicrons
and has no drug therapy available to lower the plasma
lipoprotein levels?
A. Type I.
B. Type II.
C. Type III.
D. Type IV.
E. Type V.

A

Correct answer = A. Type I hyperlipidemia (hyperchylomicronemia)
is treated with a low-fat diet. No drug therapy is
effective for this disorder.

62
Q

23.3 Which one of the following drugs decreases cholesterol
synthesis by inhibiting the enzyme 3-hydroxy-3-
methylglutaryl coenzyme A reductase?
A. Fenofibrate.
B. Niacin.
C. Cholestyramine.
D. Lovastatin.
E. Gemfibrozil.

A

Correct answer = D. Lovastatin decreases cholesterol synthesis
by inhibiting HMG CoA reductase. Fenofibrate and
gemfibrozil increase the activity of lipoprotein lipase, thereby
increasing the removal of VLDL from plasma. Niacin inhibits
lipolysis in adipose tissue, thus eliminating the building
blocks needed by the liver to produce triglycerides and,
therefore, VLDL. Cholestyramine lowers the amount of bile
acids returning to the liver via the enterohepatic circulation

63
Q

23.4 Which one of the following drugs causes a decrease in
liver triglyceride synthesis by limiting available free fatty
acids needed as building blocks for this pathway?
A. Niacin.
B. Fenofibrate.
C. Cholestyramine.
D. Gemfibrozil.
E. Lovastatin.

A

Correct answer = A. At gram doses, niacin strongly inhibits
lipolysis in adipose tissue—the primary producer of circulating
free fatty acids. The liver normally utilizes these
circulating fatty acids as a major precursor for triglyceride
synthesis. Thus, niacin causes a decrease in liver triglyceride
synthesis, which is required for VLDL production. The
other choices do not inhibit lipolysis in adipose tissue.

64
Q

23.5 Which one of the following drugs binds bile acids in the
intestine, thus preventing their return to the liver via the
enterohepatic circulation?
A. Niacin.
B. Fenofibrate.
C. Cholestyramine.
D. Fluvastatin.
E. Lovastatin

A

Correct answer = C. Cholestyramine is an anion-exchange
resin that binds negatively charged bile acids and bile
salts in the small intestine. The resin/bile acid complex is
excreted in the feces, thus preventing the bile acids from
returning to the liver by the enterohepatic circulation. The
other choices do not bind intestinal bile acids

65
Q

23.6 JS is a 65-year-old man who presents to his physician
for management of hyperlipidemia. His most recent
lipid panel reveals an LDL cholesterol level of 165 mg/
dL. His physician wishes to begin treatment to lower his
LDL cholesterol levels. Which of the following therapies
is the best option to lower JS’s LDL cholesterol levels?
A. Fenofibrate.
B. Colesevelam.
C. Niacin.
D. Simvastatin.
E. Ezetimibe.

A

Correct answer = D. Simvastatin, an HMG CoA reductase
inhibitor (statin), is the most effective option for lowering
LDL cholesterol, achieving reductions of 30% to 41% from
baseline levels. Fenofibrate and niacin are more effective
at lowering triglyceride levels or raising HDL levels (niacin).
Colesevelam can reduce LDL levels but not as effectively as
statins. Ezetimibe lowers LDL levels modestly compared to
the LDL reduction achieved by statins

66
Q

23.7 WW is a 62-year-old female with hyperlipidemia and
hypothyroidism. Her current medications include
cholestyramine and levothyroxine (thyroid hormone).
What advice would you give to WW to avoid a
drug interaction between her cholestyramine and
levothyroxine?
A. Stop taking the levothyroxine as it can interact
with cholestyramine.
B. Take levothyroxine 1 hour before cholestyramine
on an empty stomach.
C. Switch cholestyramine to colesevelam as this will
eliminate the interaction.
D. Switch cholestyramine to colestipol as this will
eliminate the interaction.
E. Take levothyroxine and cholestyramine at the
same time to minimize the interaction.

A

Correct answer = B. Cholestyramine and the bile acid resins
can bind several medications causing decreased absorption.
Cholestyramine can decrease absorption of medications
such as levothyroxine. Taking levothyroxine 1 hour
before or 4 to 6 hours after cholestyramine can help to avoid
this interaction. Choices C and D are incorrect, as all bile
acid resins cause this interaction. Choice A is incorrect, as
this patient should not stop her thyroid medication. Choice
E will worsen this drug interaction.

67
Q

23.8 AJ is a 42-year-old man who was started on niacin
sustained-release tablets 2 weeks ago for elevated
triglycerides and low HDL levels. He is complaining
of an uncomfortable flushing and itchy feeling that he
thinks is related to the niacin. Which of the following
options can help AJ manage this adverse effect of
niacin therapy?
A. Administer aspirin 30 minutes prior to taking
niacin.
B. Administer aspirin 30 minutes after taking niacin.
C. Increase the dose of niacin SR to 1000 mg.
D. Continue the current dose of niacin.
E. Change the sustained-release niacin to
immediate-release niacin.

A

Correct answer = A. Flushing associated with niacin is
prostaglandin mediated; therefore, use of aspirin (a prostaglandin
inhibitor) can help to minimize this adverse effect.
It must be administered 30 minutes prior to the dose of
the niacin; therefore, choice B is incorrect. Increasing the
dose of niacin is likely to increase these complaints; therefore,
choice C is incorrect. Continuing the current dose is
unlikely to relieve these complaints, which are bothersome
to AJ. The sustained-release formulation of niacin has less
incidence of flushing versus that of the immediate release;
therefore, choice E is incorrect.

68
Q

CN is a 72-year-old male who is treated for hyperlipidemia
with high-dose atorvastatin for the past 6 months. He
also has a history of renal insufficiency. His most
recent lipid panel shows an LDL cholesterol level
of 131 mg/dL, triglycerides of 510 mg/dL, and HDL
cholesterol of 32 mg/dL. His physician wishes to add
an additional agent for his hyperlipidemia. Which of
the following choices is the best option to address
CN’s dyslipidemia?
A. Fenofibrate.
B. Niacin.
C. Colesevelam.
D. Gemfibrozil.
E. Ezetimibe.

A

Correct answer = B. This patient has significantly elevated
triglycerides and low HDL. Niacin can lower triglycerides by
35% to 50% and also raise HDL levels. The fibrates (fenofibrate
and gemfibrozil) should not be used due to CN’s
history of renal insufficiency. Use of colesevelam is contraindicated
because triglycerides are greater than 400 mg/dL.
Ezetimibe can further lower LDL cholesterol but has modest
effects on triglycerides versus niacin.

69
Q

23.10 Which of the following patient populations is more
likely to experience myalgia (muscle pain) or myopathy
with use of HMG CoA reductase inhibitors?
A. Patients with diabetes mellitus.
B. Patients with renal insufficiency.
C. Patients with gout.
D. Patients with hypertriglyceridemia.
E. Patients taking warfarin (blood thinner

A

Correct answer = B. Patients with a history of renal insufficiency
have a higher incidence of developing myalgias,
myopathy, and rhabdomyolysis with use of HMG CoA
reductase inhibitors (statins), especially with those that are
renally eliminated as drug accumulation can occur. The
other populations have not been reported to have a higher
incidence of this adverse effect with HMG CoA reductase
inhibitors.